Calculating Flux: Homework Statement

  • Thread starter Thread starter oteggis
  • Start date Start date
  • Tags Tags
    Flux
Click For Summary
The discussion revolves around calculating the flux of a vector field F through a solid R defined within a cylinder and bounded by specific planes. The calculations involve applying the Divergence Theorem to relate the surface integrals to volume integrals. Key results include the flux through the top surface yielding 1200π and the divergence of F calculated as -3(x^2+y^2-2z). There is also clarification on the outward normal vector, which is crucial for accurate application of the theorem. Overall, the thread emphasizes the importance of proper notation and understanding of vector calculus concepts in solving the problem.
oteggis
Messages
9
Reaction score
0

Homework Statement


Let S be the surface of a solid R , which lies inside the cylinder:
##x^2+y^2=16##
and between the plane

where x=0 and z=5

There is also defined a vector field F by:
##\begin{align}F(x,y)=(-x^3i-y^3j+3z^2k)\end{align}##

(a) Calculate : $$\iint_{T} F.\hat n\mathrm dS$$with T = {##(x,y,5)∈ℝ | x^2+y^2≤16##}(b) Calculate DivF and $$\iint_{S} F.\hat n\mathrm dS$$

with n the outward pointing unit normal.

(c) Calculate: $$\iint_{V} F.\hat n\mathrm dS$$

with V = {##(x,y,z)∈ℝ |x^2+y^2≤16 and 0≤z≤5##} and the unit normal ##\hat n\ ##points out of the solid R

Homework Equations


$$\iint_{V} F.\hat n\mathrm dS$$

The Attempt at a Solution


(a) On the top surface of the Cylinder z = 5, ##\hat n##= ##\hat k##
##F.\hat n = [-x^3i +-y^3j +3z^2k].[k]=3z^2 ##
##\iint_{S} F.\hat n \mathrm dS = \iint_{S} 3z^2\mathrm dS##
##\iint_{S} F.\hat n \mathrm dS = \iint_{S} 3(5^2)\mathrm dS##
##\iint_{S} F.\hat n \mathrm dS = 75\iint_{S}\mathrm dS##
The area enclosed by the circle is ##16\pi##
since the radius of the circle is 4.
Therefore
##\iint_{S} F.\hat n \mathrm dS = 75(16\pi) = 1200\pi ##(b) ##DivF = \nabla.F = [i\frac{\partial }{\partial x}+ j\frac{\partial }{\partial y} +k\frac{\partial }{\partial z}].[-x^3i +-y^3j +3z^2k]= -3x^2-3y^2 +6z##
DivF = ∇.F = -3(x^2+y^2-2z)
##\iint_{S} F.\hat n \mathrm dS =\iiint_{V} \operatorname{div} F dV -\iint_{S_1} F.\hat n \mathrm dS##
## \iint_{S_1} F.\hat n \mathrm dS = 0 ## since z=0, then
## \iint_{S} F.\hat n \mathrm dS = \iiint_{V} \operatorname{div} F dV ##**How do I get ##\hat n## in this case?**

(c) From my understanding, I have to use Divergence Theorem here
$$\iint_{V} F.\hat n\mathrm dS = \iiint_{V} \nabla.F \mathrm dV $$
$$\iint_{V} F.\hat n\mathrm dS = \iiint_{V} -3(x^2+y^2-2z) \mathrm dV $$
$$\iint_{V} F.\hat n\mathrm dS = -3\iiint_{V} (x^2+y^2-2z) \mathrm dV $$

Using Cylindrical coordinates
$$\iint_{V} F.\hat n\mathrm dS = -3\iiint_{V} [(r^2\cos^2\theta+r^2\sin^2\theta-2z)] \mathrm rdzdrd\theta $$
$$\iint_{V} F.\hat n\mathrm dS = -3\int_0^{2\pi} \int_0^4 \int_0^5 [(r^2\cos^2\theta+r^2\sin^2\theta-2z)] \mathrm rdzdrd\theta $$
$$\iint_{V} F.\hat n\mathrm dS = 432\pi $$
 
Last edited:
Physics news on Phys.org
Hello oteggis,

Could you read what you posted and fix the typesetting ? for in-line math you want to use ## ... ## delimiters and for displayed math $$ ... $$

I take it the solid ##R## completely fills the cylinder ? ('lies inside' can apply to a very tiny fraction of the space). And not ##x=0## but ##z=0## ?
 
On the curved cylinder surface, $$\hat{n}=\hat{r}=\frac{x\hat{i}+y\hat{j}}{4}$$
 
Chestermiller said:
On the curved cylinder surface, $$\hat{n}=\hat{r}=\frac{x\hat{i}+y\hat{j}}{4}$$
so in Polar coordinates: ##\hat n## = ##cos\theta i + sin\theta j##
But I got ##\hat n## = ##-cos\theta i-sin\theta j## instead
 
oteggis said:
so in Polar coordinates: ##\hat n## = ##cos\theta i + sin\theta j##
But I got ##\hat n## = ##-cos\theta i-sin\theta j## instead
That's the inward-directed normal, not the outward normal required by the divergence theorem.
 
Question: A clock's minute hand has length 4 and its hour hand has length 3. What is the distance between the tips at the moment when it is increasing most rapidly?(Putnam Exam Question) Answer: Making assumption that both the hands moves at constant angular velocities, the answer is ## \sqrt{7} .## But don't you think this assumption is somewhat doubtful and wrong?

Similar threads

  • · Replies 6 ·
Replies
6
Views
2K
  • · Replies 3 ·
Replies
3
Views
2K
  • · Replies 4 ·
Replies
4
Views
2K
  • · Replies 11 ·
Replies
11
Views
2K
  • · Replies 4 ·
Replies
4
Views
2K
  • · Replies 1 ·
Replies
1
Views
5K
  • · Replies 6 ·
Replies
6
Views
3K
  • · Replies 6 ·
Replies
6
Views
2K
Replies
8
Views
3K
  • · Replies 7 ·
Replies
7
Views
4K